Jump to content
Urch Forums

romit_z

1st Level
  • Posts

    105
  • Joined

Converted

  • My Tests
    No

romit_z's Achievements

Newbie

Newbie (1/14)

1

Reputation

  1. I too selected C but the answer in the Sets is A. I feel the answer is wrong as without both the statements it is not possible to answer the question
  2. Thanks Luca, that was a perfect explanation. I did not understrand the question, but the explanation made it clear.
  3. Thanks Luca, that was a perfect explanation. I did not understrand the question, but the explanation made it clear.
  4. I am not sure why my answer is incorrect. OA to follow
  5. I am confused with this percentage question :(
  6. "shoppers at ShopperKing would not save money overall by shopping at FoodBasket instead" . This can be substantiated only by the fact that shoppers shop for particular items at a store where those items are cheaper and people who are buying at ShopperKing are buying items that are cheaper there and wont get them cheaper at FoodBasket ... A clearly says that.
  7. Use the negation approach. If not D not C, not C not B, Not B not A. Answer is C.
  8. i feel E. Cause if the crabs shed there shell several times so then they won't have the barnacles attached to them even though they were attached to them at the first instance.
  9. 3 is the answer as it weakens the conclusion that "hippocampi had lost cell mass as a result of trauma. " It says the reduced volume of hippocampi makes an individual more susceptible to post-traumatic stress symptoms.
  10. Real tough one, I would go for E. E tells that manatees are unable to distinguish sounds of different pitch and therefore the conclusion that by lowering of speed will cause less deaths is weakened.A manatee is not able to distinguish between hig pitch or low pitch . Not sure though...what is OA?
  11. E supports why the course of action suggested might not be a good way of tackling this situation.
  12. I dont know the OA for this. A is very attractive choice but what I feel is it brings a new set of reasoning out of scope of the passage. The arguement is "the austere budgets during Governor Verdant’s term have caused the slowdown ". We cannot have a direct correlation between budget spending and inflation. I feel C) which says every year the budget has shown some increase is the choice which most weakens the arguement. Any more views around?
  13. I would go for E. The arguement is that the new instrument megatelescope has a potential like earlier research areas to develop into a useful application. But if we can prove that it is not worthy of the same comparison the arguement is weakened. E clearly does that.
  14. While Governor Verdant has been in office, the state’s budget has increased by an average of 6 percent each year. While the previous governor was in office, the state’s budget increased by an average of 11.5 percent each year. Obviously, the austere budgets during Governor Verdant’s term have caused the slowdown in the growth in state spending. Which of the following, if true, would most seriously weaken the conclusion drawn above? (A) The rate of inflation in the state averaged 10 percent each year during the previous governor’s term in office and 3 percent each year during Verdant’s term. (B) Both federal and state income tax rates have been lowered considerably during Verdant’s term in office. © In each year of Verdant’s term in office, the state’s budget has shown some increase in spending over the previous year. (D) During Verdant’s term in office, the state has either discontinued or begun to charge private citizens for numerous services that the state offered free to citizens during the previous governor’s term. (E) During the previous governor’s term in office, the state introduced several so-called “austerity” budgets intended to reduce the growth in state spending.
  15. Thanks, your answers are correct and explanation fantastic.
×
×
  • Create New...